AMC 10 Daily Practice - Sequences

Complete problem set with solutions and individual problem pages

Problem 1 Easy

S_{n} is the sum of the first n terms of the arithmetic sequence \left\{a_{n}\right\}. If d=2 and S_{5}=15, then a_{1}=            .

  • A.

    -1

  • B.

    -2

  • C.

    -3

  • D.

    -4

Answer:A

S_{5}=5a_{1}+\frac{5\times 4}{2}\times 2=15,

then a_{1}=-1.

Link Problem
Problem 2 Medium

Consider sequences of positive integers for which both the following conditions are true:

(a) each term after the second term is the sum of the two preceding terms;

(b) the eighth term is 260.

How many such sequences are there?

  • A.

    1

  • B.

    2

  • C.

    3

  • D.

    4

  • E.

    5

Answer:B

Let a and b be the first and second terms respectively. Then the first eight terms are

a,b,a+b,a+2b,2a+3b,3a+5b,5a+8b,8a+13b

Hence we seek solutions to the equation

8a+13b=260,  (1)

where a and b are positive integers, since any such solution will generate a sequence of positive integers of the required sort.

Now 8a=13(20-13b=260-b), which is a multiple of 13. Therefore 8a is a multiple of 13, so that a is a multiple of 13.

Let a = 13k, where k is a positive integer. Then equation (1) becomes

8 \times 13k+13k=260 ,

so that

8k+b=20

Since b and k are positive integers there are therefore only two possible values for k, namely 1 and 2. When k=1, we have a =13 and b=12. When k=2, we have a = 26 and b = 4. Hence, there are two possible sequences.

Link Problem
Problem 3 Easy

Define a sequence recursively by x_0=5 andx_{n+1}=\frac{x_n^2+5x_n+4}{x_n+6}for all nonnegative integers n. Let m be the least positive integer such that x_m\leq 4+\frac{1}{2^{20}}. In which of the following intervals does m lie? (2019 AMC 10B Problems, Question #24)

  • A.

    [9,26]

  • B.

    [27,80]

  • C.

    [81,242]

  • D.

    [243,728]

  • E.

    [729,\infty)

Answer:C

The condition where x_m\leq 4+\frac{1}{2^{20}} gives the motivation to make a substitution to change the equilibrium from 4 to 0. We can substitute x_n = y_n + 4 to achieve that. Now, we need to find the smallest value of m such that y_m\leq \frac{1}{2^{20}} given that y_0 = 1.

 

Factoring the recursion x_{n+1} = \frac{x_n^2 + 5x_n+4}{x_n + 6}, we get:

 

x_{n+1}=\dfrac{(x_n + 4)(x_n + 1)}{x_n + 6} \Rightarrow y_{n+1}+4=\dfrac{(y_n+8)(y_n+5)}{y_n+10}

 

y_{n+1}+4=\dfrac{y_n^2+13y_n+40}{y_n+10} = \dfrac{y_n^2+9y_n +(4y_n+40)}{y_n+10}

 

y_{n+1}+4=\dfrac{y_n^2+9y_n}{y_n+10} + 4

 

y_{n+1}=\dfrac{y_n^2+9y_n}{y_n+10}.

 

Using wishful thinking, we can simplify the recursion as follows:

 

y_{n+1} = \frac{y_n^2 + 9y_n + y_n - y_n}{y_n + 10}

 

y_{n+1} = \frac{y_n(y_n + 10) - y_n}{y_n + 10}

 

y_{n+1} = y_n - \frac{y_n}{y_n + 10}

 

y_{n+1} = y_n\left(1 - \frac{1}{y_n + 10}\right).

 

The recursion looks like a geometric sequence with the ratio changing slightly after each term. Notice from the recursion that the y_n sequence is strictly decreasing, so all the terms after y_0 will be less than 1. Also, notice that all the terms in sequence will be positive. Both of these can be proven by induction.

 

With both of those observations in mind, \frac{9}{10} < 1 - \frac{1}{y_n + 10} \leq \frac{10}{11}. Combining this with the fact that the recursion resembles a geometric sequence, we conclude that \left(\frac{9}{10}\right)^n < y_n \leq \left(\frac{10}{11}\right)^n.

 

\frac{9}{10} is approximately equal to \frac{10}{11} and the ranges that the answer choices give us are generous, so we should use either \frac{9}{10} or \frac{10}{11} to find a rough estimate for m.

 

Since \dfrac{1}{2}=0.5, that means \frac{1}{\sqrt{2}}=2^{-\frac{1}{2}} \approx 0.7. Additionally, \left(\frac{9}{10}\right)^3=0.729

 

Therefore, we can estimate that 2^{-\frac{1}{2}} < y_3.

 

Raising both sides to the 40th power, we get 2^{-20} < (y_3)^{40}

 

But y_3 = (y_0)^3, so 2^{-20} < (y_0)^{120} and therefore, 2^{-20} < y_{120}.

 

This tells us that m is somewhere around 120, so our answer is \boxed{\textbf{(C) } [81,242]}.

Link Problem
Table of Contents
  • 1
  • 2
  • 3